Q7

 
fyami001
Thanks Received: 0
Vinny Gambini
Vinny Gambini
 
Posts: 19
Joined: May 08th, 2011
 
 
 

Q7

by fyami001 Fri Aug 12, 2011 6:26 pm

Hello!
I have a personal vendetta against CBT questions. they are so tricky! My only way to get vengeance is to get them right lol and I can't figure this one out.

My issue here is that to me it seems like answer choice "C
is also true. I am basing this off the diagram that was posted.

In Case 2 where G is ranked as the best or #1, I, K-F must fit into the slots 3,4,or 5 in no particular order, except that K cannot go 5. Why then, do we have to throw out choice "c" where I ranks better than F out and choose choice "E" instead?

Thanks in advance for helping me achieve vengeance Manhattan LSAT staff!:P
 
timmydoeslsat
Thanks Received: 887
Atticus Finch
Atticus Finch
 
Posts: 1136
Joined: June 20th, 2011
 
 
trophy
Most Thanked
trophy
First Responder
 

Re: Q7

by timmydoeslsat Fri Aug 12, 2011 11:17 pm

The answer is C!

You will learn to love could be true questions. The right answer will be something that could be true, while the other four choices will be MUST BE FALSE situations.

In other questions, you could use the four answers from this question that must be false to attack other problems if needed.

Let me show you why C is correct.

A) G ranks better than M

Well we know that L is second in our diagram and that I will be tested. So we know we have two variables already in the five variable slots. Bringing along G and M gives us 4. We have room for one more. Bringing in M means we have F and H. Too many variables that must be tested. This is something that must be false.


B) H ranks better than F

For H to rank better than F, we would have to have G be first. Either F or G must be first. To have a situation where H ranks better than F would require us to have G first.

G is first, L is second, and I is used. Now we know that to make this scenario work, we need H to be ahead of F. But notice that since G is now in our scenario, G and H are both in our scenario. The rules require us to have H rank better than G if both are used. This is impossible with G forcing to be first. This is something that must be false.


C) I ranks better than F

I is essentially a free agent in this game. All we know about it is that it has to be within the five medications being tested. We know that for I to be ahead of F, G would have to go first.

So we have

1) G 2) L 3) ___ 4) ___ 5) ___

We know we want to have it to where I is ahead of F, so I - F

We can smell that this can work! We just have to be careful about who we bring in to be this last variable. Notice that only one more variable can fit.

We do not want to bring in H, because G is first and there is no way that H could be ahead of it. There is no way we could bring in M because that would bring in too many variables to fit within the 5, as it would bring in F and H. (Plus, it brings in H!)

So those will be the two that are out! There are 7 variables and 5 are selected. That means 2 are not selected. We have both variables that must be out, M and H.

Only K is left. We know that to bring in K in this scenario we have to make sure that K is ahead of F. No problem!

1) G 2) L 3) ___ 4) ___ 5) ___

We know I - F.....and know we know K...F

That means that

1) G 2) L 3) K/I 4) I/K 5) F

K and I can go in either three or four.

After knowing that this can work, circle and move on. Do not go through the others. I will go through them to be thorough.


D) K ranks better than G

That would require having F first.

1) F 2) L 3) ___ 4) ___ 5) ___

Notice that obviously F is now in the mix with K, and K must be ranked better. Impossible. Must be false.


E) M ranks better than G

We know that I must be tested, L is second, and these two would make our variables tested at 4. Room for one more. M brings in 2 more! Too many! Must be false!